Difference between revisions of "2024 AMC 8 Problems"

(Replaced content with "skibidi toilet GGYATT")
(Tag: Replaced)
(Undo revision 212804 by Sdlighe (talk))
(Tag: Undo)
Line 1: Line 1:
skibidi toilet GGYATT
+
{{AMC8 Problems|year=2024|}}
 +
==Problem 1==
 +
What is the ones digit of <cmath>222,222-22,222-2,222-222-22-2?</cmath>
 +
<math>\textbf{(A) } 0\qquad\textbf{(B) } 2\qquad\textbf{(C) } 4\qquad\textbf{(D) } 6\qquad\textbf{(E) } 8</math>
 +
 
 +
[[2024 AMC 8 Problems/Problem 1|Solution]]
 +
 
 +
==Problem 2==
 +
What is the value of this expression in decimal form?
 +
<cmath>\frac{44}{11} + \frac{110}{44} + \frac{44}{1100}</cmath>
 +
 
 +
<math>\textbf{(A) } 6.4\qquad\textbf{(B) } 6.504\qquad\textbf{(C) } 6.54\qquad\textbf{(D) } 6.9\qquad\textbf{(E) } 6.94</math>
 +
 
 +
[[2024 AMC 8 Problems/Problem 2|Solution]]
 +
 
 +
==Problem 3==
 +
 
 +
==Problem 4==
 +
When Yunji added all the integers from <math>1</math> to <math>9</math>, she mistakenly left out a number. Her incorrect sum turned out to be a square number. What number did Yunji leave out?
 +
 
 +
<math>\textbf{(A) } 5\qquad\textbf{(B) } 6\qquad\textbf{(C) } 7\qquad\textbf{(D) } 8\qquad\textbf{(E) } 9</math>
 +
 
 +
[[2024 AMC 8 Problems/Problem 4|Solution]]
 +
 
 +
==Problem 5==
 +
Aaliyah rolls two standard 6-sided dice. She notices that the product of the two numbers rolled is a multiple of <math>6</math>. Which of the following integers cannot be the sum of the two numbers?
 +
 
 +
<math>\textbf{(A) } 5\qquad\textbf{(B) } 6\qquad\textbf{(C) } 7\qquad\textbf{(D) } 8\qquad\textbf{(E) } 9</math>
 +
 
 +
[[2024 AMC 8 Problems/Problem 5|Solution]]
 +
 
 +
==Problem 6==
 +
 
 +
==Problem 7==
 +
 
 +
==Problem 8==
 +
On Monday Taye has \$2. Every day, he either gains \$3 or doubles the amount of money he had on the previous day. How many different dollar amounts could Taye have on Thursday, 3 days later?
 +
 
 +
<math>\textbf{(A) } 3\qquad\textbf{(B) } 4\qquad\textbf{(C) } 5\qquad\textbf{(D) } 6\qquad\textbf{(E) } 7</math>
 +
 
 +
[[2024 AMC 8 Problems/Problem 8|Solution]]
 +
 
 +
==Problem 10==
 +
 
 +
==Problem 11==
 +
 
 +
==Problem 12==
 +
 
 +
==Problem 13==
 +
 
 +
==Problem 14==
 +
 
 +
==Problem 15==
 +
 
 +
Let the letters <math>F</math>,<math>L</math>,<math>Y</math>,<math>B</math>,<math>U</math>,<math>G</math> represent distinct digits. Suppose <math>\underline{F}~\underline{L}~\underline{Y}~\underline{F}~\underline{L}~\underline{Y}</math> is the greatest number that satisfies the equation
 +
 
 +
<cmath>8\cdot\underline{F}~\underline{L}~\underline{Y}~\underline{F}~\underline{L}~\underline{Y}=\underline{B}~\underline{U}~\underline{G}~\underline{B}~\underline{U}~\underline{G}.</cmath>
 +
 
 +
What is the value of <math>\underline{F}~\underline{L}~\underline{Y}+\underline{B}~\underline{U}~\underline{G}</math>?
 +
 
 +
<math>\textbf{(A)}\ 1089 \qquad \textbf{(B)}\ 1098 \qquad \textbf{(C)}\ 1107 \qquad \textbf{(D)}\ 1116 \qquad \textbf{(E)}\ 1125</math>
 +
 
 +
[[2024 AMC 8 Problems/Problem 15|Solution]]
 +
 
 +
==Problem 16==
 +
 
 +
==Problem 17==
 +
 
 +
==Problem 18==
 +
 
 +
==Problem 19==
 +
 
 +
==Problem 20==
 +
 
 +
==Problem 21==
 +
A group of frogs (called an army) is living in a tree. A frog turns green when in the shade and turns yellow
 +
when in the sun. Initially, the ratio of green to yellow frogs was <math>3 : 1</math>. Then <math>3</math> green frogs moved to the
 +
sunny side and <math>5</math> yellow frogs moved to the shady side. Now the ratio is <math>4 : 1</math>. What is the difference
 +
between the number of green frogs and the number of yellow frogs now?
 +
 
 +
<math>\textbf{(A) } 10\qquad\textbf{(B) } 12\qquad\textbf{(C) } 16\qquad\textbf{(D) } 20\qquad\textbf{(E) } 24</math>
 +
 
 +
[[2024 AMC 8 Problems/Problem 21|Solution]]
 +
 
 +
==Problem 22==
 +
 
 +
==Problem 23==
 +
 
 +
==Problem 24==
 +
 
 +
==Problem 25==
 +
 
 +
==See Also==
 +
{{AMC8 box|year=2024|before=[[2023 AMC 8 Problems|2023 AMC 8]]|after=[[2025 AMC 8 Problems|2025 AMC 8]]}}
 +
* [[AMC 8]]
 +
* [[AMC 8 Problems and Solutions]]
 +
* [[Mathematics competition resources|Mathematics Competition Resources]]

Revision as of 16:49, 25 January 2024

2024 AMC 8 (Answer Key)
Printable versions: WikiAoPS ResourcesPDF

Instructions

  1. This is a 25-question, multiple choice test. Each question is followed by answers marked A, B, C, D and E. Only one of these is correct.
  2. You will receive 1 point for each correct answer. There is no penalty for wrong answers.
  3. No aids are permitted other than plain scratch paper, writing utensils, ruler, and erasers. In particular, graph paper, compass, protractor, calculators, computers, smartwatches, and smartphones are not permitted. Rules
  4. Figures are not necessarily drawn to scale.
  5. You will have 40 minutes working time to complete the test.
1 2 3 4 5 6 7 8 9 10 11 12 13 14 15 16 17 18 19 20 21 22 23 24 25

Problem 1

What is the ones digit of \[222,222-22,222-2,222-222-22-2?\] $\textbf{(A) } 0\qquad\textbf{(B) } 2\qquad\textbf{(C) } 4\qquad\textbf{(D) } 6\qquad\textbf{(E) } 8$

Solution

Problem 2

What is the value of this expression in decimal form? \[\frac{44}{11} + \frac{110}{44} + \frac{44}{1100}\]

$\textbf{(A) } 6.4\qquad\textbf{(B) } 6.504\qquad\textbf{(C) } 6.54\qquad\textbf{(D) } 6.9\qquad\textbf{(E) } 6.94$

Solution

Problem 3

Problem 4

When Yunji added all the integers from $1$ to $9$, she mistakenly left out a number. Her incorrect sum turned out to be a square number. What number did Yunji leave out?

$\textbf{(A) } 5\qquad\textbf{(B) } 6\qquad\textbf{(C) } 7\qquad\textbf{(D) } 8\qquad\textbf{(E) } 9$

Solution

Problem 5

Aaliyah rolls two standard 6-sided dice. She notices that the product of the two numbers rolled is a multiple of $6$. Which of the following integers cannot be the sum of the two numbers?

$\textbf{(A) } 5\qquad\textbf{(B) } 6\qquad\textbf{(C) } 7\qquad\textbf{(D) } 8\qquad\textbf{(E) } 9$

Solution

Problem 6

Problem 7

Problem 8

On Monday Taye has $2. Every day, he either gains $3 or doubles the amount of money he had on the previous day. How many different dollar amounts could Taye have on Thursday, 3 days later?

$\textbf{(A) } 3\qquad\textbf{(B) } 4\qquad\textbf{(C) } 5\qquad\textbf{(D) } 6\qquad\textbf{(E) } 7$

Solution

Problem 10

Problem 11

Problem 12

Problem 13

Problem 14

Problem 15

Let the letters $F$,$L$,$Y$,$B$,$U$,$G$ represent distinct digits. Suppose $\underline{F}~\underline{L}~\underline{Y}~\underline{F}~\underline{L}~\underline{Y}$ is the greatest number that satisfies the equation

\[8\cdot\underline{F}~\underline{L}~\underline{Y}~\underline{F}~\underline{L}~\underline{Y}=\underline{B}~\underline{U}~\underline{G}~\underline{B}~\underline{U}~\underline{G}.\]

What is the value of $\underline{F}~\underline{L}~\underline{Y}+\underline{B}~\underline{U}~\underline{G}$?

$\textbf{(A)}\ 1089 \qquad \textbf{(B)}\ 1098 \qquad \textbf{(C)}\ 1107 \qquad \textbf{(D)}\ 1116 \qquad \textbf{(E)}\ 1125$

Solution

Problem 16

Problem 17

Problem 18

Problem 19

Problem 20

Problem 21

A group of frogs (called an army) is living in a tree. A frog turns green when in the shade and turns yellow when in the sun. Initially, the ratio of green to yellow frogs was $3 : 1$. Then $3$ green frogs moved to the sunny side and $5$ yellow frogs moved to the shady side. Now the ratio is $4 : 1$. What is the difference between the number of green frogs and the number of yellow frogs now?

$\textbf{(A) } 10\qquad\textbf{(B) } 12\qquad\textbf{(C) } 16\qquad\textbf{(D) } 20\qquad\textbf{(E) } 24$

Solution

Problem 22

Problem 23

Problem 24

Problem 25

See Also

2024 AMC 8 (ProblemsAnswer KeyResources)
Preceded by
2023 AMC 8
Followed by
2025 AMC 8
1 2 3 4 5 6 7 8 9 10 11 12 13 14 15 16 17 18 19 20 21 22 23 24 25
All AJHSME/AMC 8 Problems and Solutions